endocrine for EOC

¡Supera tus tareas y exámenes ahora con Quizwiz!

High output heart failure dx: thyroid storm. Thyroid storm is a rare, life-threatening hypermetabolic state caused by severe thyrotoxicosis. The most common precipitating factor is an infection. A well-recognized and important complication of thyroid storm is high output heart failure. What is the expected appearance of the chest radiograph in a patient with high output heart failure? Answer: Bilateral pulmonary edema with a normally-sized heart. Adrenergic blockade is the most important initial component of therapy. Propranolol,

A 42-year-old man presents to the ED with racing heart rate and difficulty breathing. On physical exam, you note bilateral ocular proptosis and 3+ non-pitting edema of the bilateral lower extremities. His electrocardiogram is shown above. Which of the following complications of this disease is this patient most likely experiencing? ACardiac tamponade BHigh output heart failure CPulmonary embolism DTension pneumothorax

An 18-year-old woman presents with fever, confusion, weight loss, and palpitations. Her vital signs are T 100.7°F, HR 140 bpm, BP 143/93 mm Hg, RR 20/min, oxygen saturation 95%, and finger stick glucose of 118 mg/dL. Physical examination reveals a thin woman who is alert and oriented x 3 with a regular, fast heart rate and brisk reflexes. Her roommate tells you the patient has been taking a friend's levothyroxine in an attempt to lose weight. What management should be initiated in addition to discontinuing the medication?

Beta-blocker dx: thyroid storm secondary to exogenous thyroid hormone abuse

A 5-month-old girl is transferred from an outside facility with concerns of bilateral ptosis, ophthalmoplegia, weak suck, mild diffuse hypotonia, and constipation that have developed and worsened over the past three days. A detailed history reveals that the infant was born at term via an uncomplicated home delivery. She has been previously healthy, growing, and normally developing. Her vaccine schedule has been delayed due to parental hesitancy. She has had no recent illness, sick contacts, or trauma. She has one healthy older brother who is homeschooled and she does not attend daycare. Her parents are gradually introducing her to new fruits and vegetables from their supply of home-canned goods, and she has thus far tolerated them well. Initial laboratory evaluation included unremarkable blood counts and liver panel. Serum chemistries were significant for a slightly elevated bicarbonate level. A capillary blood gas revealed a respiratory acidosis with partial metabolic compensation. A non-contrast CT of the brain was normal and subsequent cerebrospinal fluid analysis was normal. Which of the following is indicated for treatment?

Botulism immune globulin Infant Botulism Patient will be an infant History of eating honey or residing near construction site Feeble cry, constipation PE will show symmetric descending paralysis (floppy baby) Most commonly caused by Clostridium botulinum Treatment is IV botulism Ig s cranial nerve abnormalities, diffuse and progressive hypotonia, and new-onset constipation are concerning for infantile botulism

A 26-year-old woman presents with a 5-month history of amenorrhea and bilateral galactorrhea. She denies the possibility of pregnancy. On physical exam, milk can be expressed from both breasts. Visual fields testing by confrontation reveals bitemporal field defects. Pelvic exam is normal and a pregnancy test is negative. After confirming the suspected diagnosis, which of the following is the most appropriate management?***** Which class of medications is most likely to cause hyperprolactinemia? A 47-year-old man presents with decreased libido and low energy levels. In addition to low testosterone levels, laboratory results show a prolactin level of 102 ng/mL. All other hormone levels tested are normal. The patient is sent for MRI of the brain, which reveals an 8 mm x 4 mm sellar mass. Which of the following is the most appropriate therapy?

Cabergoline dx: prolactinoma The diagnosis of prolactinoma is confirmed with a prolactin level > 200 ng/mL. Magnetic resonance imaging is the imaging test of choice for hyperprolactinemia that has no known pharmacologic or physiologic causes. Dopamine agonists reduce the size of prolactinomas in 90% of patients. For decreasing serum prolactin level, cabergoline is considered superior to bromocriptine. Surgery: indicated in adenomas refractory to medical management or with compressive effects (e.g., visual loss) Prolactinoma is the most common functioning pituitary tumor Which class of medications is most likely to cause hyperprolactinemia? Answer: Antipsychotics. Hyperprolactinemia Sx: infertility, galactorrhea, amenorrhea, headache, bitemporal hemianopsia Labs: elevated prolactin levels (> 200 ng/mL typically due to prolactinoma) MRI: may show a sellar mass Most commonly caused by pituitary adenoma Other causes: nipple stimulation, drugs (antipsychotic, antidepressant, antiemetic), hypothyroid, CKD, cirrhosis TreatmentFirst line: dopamine agonists (e.g., cabergoline, bromocriptine)Surgery: indicated in adenomas refractory to medical management or with compressive effects (e.g., visual loss) Prolactinoma is the most common functioning pituitary tumor

Thyroid storm is a severe variant of thyrotoxicosis. Which of the following is the most appropriate way to diagnose it?

Clinical findings What is the mortality rate of thyroid storm? Answer: 8-25%.

68-year-old woman is diagnosed with hyperparathyroidism. Both her serum calcium level and parathyroid hormone level are elevated. Which of the following findings would be most consistent with the diagnosis?

Decreased serum phosphate level

Which of the following clinical findings can best differentiate Graves disease from other causes of hyperthyroidism?

Exophthalmos What is the cause of pretibial myxedema seen in Grave's disease? Answer: It is the result of accumulation and infiltration of hydrophilic glycosaminoglycans in the dermis.

A 30-year-old man presents with progressive memory impairment and mental fatigue after sustaining head trauma during a motorcycle accident. He also reports significant weight gain even though his diet is unchanged. Physical exam reveals generalized muscle weakness and a loss of muscle bulk but no dry mucous membranes or peripheral edema. A basic metabolic panel is normal. Electrodiagnostic testing shows no evidence of myopathy or neuropathy. A DEXA scan uncovers the presence of low bone mineral density. Which of the following serum laboratory tests would establish the diagnosis?

Insulin-like growth factor-1 Growth Hormone Deficiency Most common cause: pituitary tumor ↓ muscle mass ↓ bone density ↑ lipids ↓ memory ↓ IGF-1 Diagnosis is usually made by testing levels of serum insulin-like growth factor-1, an insulin tolerance test or a GHRH-arginine test

A 27-year-old man presents with a severe headache and blurred vision. Physical examination reveals bitemporal hemianopsia and an inability to abduct the left eye. What diagnostic modality will most likely yield a diagnosis?

MRI of the brain Hypopituitarism Adults: nonfunctioning adenoma most common cause Children: craniopharyngioma most common cause Sheehan syndrome: sudden loss of lactation ApoplexyHemorrhage into preexisting adenomaHA, AMS, visual dysfunction ​Empty sella syndrome: obese + HTN What is the most immediate life-threatening complication of pituitary apoplexy? Answer: Adrenal insufficiency.

A seven-year-old boy has redeveloped nocturnal enuresis over the past month. His family denies new stressors, but they have noted increased frequency of urination during the daytime. He has lost three pounds since his last visit six months ago. He appears thin but well hydrated. Which of the following is the most appropriate next step?

Obtain a blood glucose measurement nocturnal enuresis, polyuria, and weight loss is suggestive of diabetes mellitus. Blood glucose measurement is the first step in diagnosis for any patient with suspected new-onset diabetes. What laboratory abnormalities would be expected on urinalysis of a patient with diabetes? Answer: Glucose, ketones, and low specific gravity.

A mother brings to you a 10-year-old girl because of poor school performance. The girl used to be an excellent student, but now she has difficulty understanding math and science concepts. The mother also complains that she moves sluggishly when given chores. The patient also complains of occasional constipation. On examination, there is dry skin, brittle hair, and enlarged and firm neck mass. Which of the following is the laboratory abnormality that would be expected in this condition?

Presence of anti-thyroid peroxidase antibodies dx: Hashimoto thyroiditis or chronic autoimmune thyroiditis. Hashimoto thyroiditis is commonly found in which gender? Answer: Women. Hashimoto Thyroiditis Most common cause of hypothyroidism Autoimmune F > M Nontender goiter, hypothyroidism, elevated anti-TPO Abs Tx: levothyroxine

A 52-year-old man with a history of hypertension presents with a concern that he's "developing breasts." On exam, glandular enlargement beneath his areola is present bilaterally in a symmetric manner. It is mildly tender. Which of the following daily medications most likely contributed to his condition?

Spironolactone

A 32-year-old man with history of hypertension and insulin dependent diabetes mellitus presents to the emergency room complaining of abdominal pain with associated nausea and vomiting. The patient reports these symptoms began about 3 hours ago. He complains of 6 out of 10 diffuse cramping abdominal pain and 3 episodes of bilious emesis. He denies fever, chills, diarrhea, and exposure to ill contacts. He ran out of his insulin 2 days ago and therefore has not been receiving his required daily doses. Vital signs include blood pressure of 160/95, heart rate of 122, respiratory rate of 22, pulse oximetry of 96% on room air, and temperature of 37.3°C. A stat finger stick reveals blood glucose of 660. What is the most appropriate initial management while bloodwork is pending?

2 L bolus of normal saline dx: Diabetic Ketoacidosis (DKA) What is the most common serum ketone measured in DKA? Answer: Beta-hydroxybutyrate

Which of the following should be administered to a symptomatic 2-week-old infant with glucose of 25 mg/dL?

2-5 mL/kg of 10% dextrose D25 (B) and D50 (C) increase the risk for rebound hypoglycemia in infants who are hyperinsulinemic. . Symptoms that suggest hypoglycemia include jitteriness, tachypnea, hypotonia, poor feeding, apnea, temperature instability, seizures, and lethargy.

An 80-year-old woman with diabetes mellitus on metformin and a history of cerebrovascular accident on low-dose aspirin presents to the emergency department by ambulance after her daughter found her difficult to arouse when checking on her this morning. The daughter states that her mother was behaving normally during her last visit 3 days ago, but she had run out of metformin. Vital signs today include a heart rate of 115 bpm, blood pressure of 125/85 mm Hg, respiratory rate of 22/min, oxygen saturation of 95% on room air, and temperature of 98.6°F. On physical examination, the patient is stuporous, responding to painful stimuli, but otherwise difficult to arouse. She has no focal numbness, focal weakness, or vision changes. A CT scan of her head shows chronic microvascular ischemic changes without acute abnormality. The patient's laboratory findings are: Glucose: 1,050 mg/dL Blood urea nitrogen: 30 mg/dL Creatinine: 1.4 mg/dL Sodium: 130 mEq/L Alanine aminotransferase: 30 U/L Aspartate aminotransferase: 32 U/L Thyroid-stimulating hormone: 2 mU/L Serum bicarbonate: 21 mEq/L Lactic acid: 2 mmol/L Arterial pH: 7.35 Urinalysis: negative leukocyte esterase, negative nitrites, trace ketones Beta-hydroxybutyrate: 0.3 mmol/L Effective serum osmolality: 330 mOsm/kg Anion gap: 10 Ammonia: 60 μg/dL Which of the following is the most likely cause of this patient's altered mental status? AHyperosmolality BHyponatremia CKetoacidosis DLactic acidosis EUremic encephalopathy

Hyperosmolality It is associated with a significantly higher mortality than diabetic ketoacidosis What is the average fluid deficit in patients with hyperosmolar hyperglycemic state? Answer: The average is 20% to 25% of total body water, or 8 to 12 L. Hyperosmolar Hyperglycemic State Patient most commonly has history of type 2 DM (has been reported in children with type 1 diabetes) AMS PE will show profound dehydration Labs will show glucose > 600 mg/dL and negative ketones Management includes IV fluids, may also need insulin or electrolyte replacement

A 16-year-old girl presents to your clinic with decreased energy, poor performance in school, and constipation for the past eight months. Her physical exam is unremarkable, but you are concerned she may have thyroid disease. Which of the following is the most appropriate next best step in management?

Obtain thyroid-stimulating hormone (TSH) and free thyroxine (FT4) levels

A 28-year-old man presents to the emergency department with lethargy and vomiting. He has a history of type I diabetes mellitus. Vital signs include heart rate 112 bpm and blood pressure 110/80 mm Hg. Laboratory analysis reveals sodium 135 mEq/L, potassium 4.0 mEq/L, chloride 100 mEq/L, bicarbonate 10 mEq/L, creatinine 1.4 mg/dL, glucose 558 mg/dL, pH 7.2. Normal saline and insulin have been ordered. Which of the following should be administered next?

Potassium chloride dx: DKA Potassium should be administered with intravenous fluids when the potassium is less than or equal to 5.0 mEq/mL. In patients where the initial potassium is low (< 3.3 mEq/L), levels could become life-threatening following the administration of insulin, therefore potassium should be vigorously repleted in concentrations of 20 to 40 mEq/L prior to initiating insulin therapy.

A 55-year-old woman presents to your office with complaints of fatigue, dry skin, constipation and weight gain. Physical exam results include a heart rate of 58 bpm and diminished deep tendon reflexes. Laboratory testing reveals an elevated TSH. Which of the following is the most appropriate next step in management?

Test serum free T4 dx: hypothyroidism,

1)What is the most common cause of acute adrenal insufficiency in the United States? Answer: Rapid withdrawal of exogenous steroids in patients on long-term therapy. 2)What is the most common metabolic problem in neonates? Answer: Hypoglycemia. 3)What structure is commonly compressed and affects vision in those with a pituitary adenoma? Answer: Optic chiasm. 4)What congenital syndrome results in hypocalcemia due to absence of the parathyroid glands? Answer: DiGeorge syndrome. 5)What finding is commonly seen with primary adrenal insufficiency but not secondary adrenal insufficiency? Skin hyperpigmentation 6)What electrocardiogram finding is classically associated with hypercalcemia? Answer: Short QT interval. Treatment: IV fluids, bisphosphonates, calcitonin 7)What recreational drug mimics the effects of pheochromocytoma? Answer: Cocaine. 8)Where are the beta cells located in the pancreas? Answer: The islets of Langerhans. 9)Question: What is Conn syndrome? Answer: Excess aldosterone produced by an adrenal adenoma. 10)A patient is diagnosed with central diabetes insipidus. Which of the following medications should you prescribe? 11)What is at risk for developing by correcting hyponatremia too rapidly? Answer: Central pontine myelinosis. 12)When the blood glucose level is approximately 200 mg/dL, the fluid should be changed to 5%dextrose in 0.45% saline to avoid iatrogenic hypoglycemia and cerebral edema, and the insulin infusion should continue at a decreased rate until ketonemia has cleared and the anion gap has normalized. 13)How much does the sodium level decrease for every 100 mg/dL of glucose above 100 mg/dL? Answer: It decreases by 1.6 mEq/L. 14). They are hesitant to start her immunizations for a number of reasons, including the fact that her 3-year-old brother is currently being evaluated for autism. Which of the following is a valid reason to delay her 2-month immunizations? Moderate acute illness without fever 15)For which vaccine is a history of egg allergy considered a precaution? Answer: Influenza. 16)For which vaccine is a history of egg allergy considered a precaution? Answer: Influenza. 17)How is the expected serum osmolality calculated? Answer: 2 x [Na+] + [BUN] / 2.8 + [glucose] / 18 + [ethanol] / 4.6. 18)Which one of the following antidiabetic drugs has a mechanism of action that works through the incretin system? Sitagliptin 19) What is a common side effect of sulfonylureas when used in elderly patients? Answer: Hypoglycemia. Glimepiride Recurrent hypoglycemic episodes in patients on sulfonylureas should be treated with octreotide, a somatostatin analog. Octreotide inhibits glucose-stimulated insulin release, thus, preventing recurrent hypoglycemia. 20)What test can help to determine if a patient is surreptitiously self-administering insulin? Answer: A low C peptide level with a high insulin level is diagnostic for factitious hypoglycemia.

facts-1

1)What finding in myxedema coma is most predictive of mortality? Answer: Severe hypothermia on presentation. 2)65-year-old woman presents to the ED with a cough and rapid heart rate. She reports a history of hyperthyroidism. On physical exam, you note tachycardia and bilateral ocular proptosis. Which of the following is the most common trigger for this patient's disease process? AInfection 3) Which complication is this patient most at risk for on her current dose of levothyroxine? atrial fibrillation and decreased bone mineral density, resulting in osteoporosis and increased risk of fractures. 4)What is the most common cause of hypothyroidism? Answer: Chronic autoimmune (Hashimoto) thyroiditis. 5) What antiarrhythmic medication can cause hyperthyroidism? Answer: Amiodarone. What are the two causes of amiodarone-induced hyperthyroidism? Answer: Destructive thyroiditis (majority) and iodine load (400 times daily requirement). 6)What is the most common clinical manifestation for a patient with a thyroid-stimulating hormone (TSH) secreting pituitary adenoma? Answer: Bitemporal hemianopsia. 7)How often should a subcentimeter nodule with suspicious characteristics be monitored? Answer: Every six to 12 months. 8)Fasting serum blood glucose > 126 mg/dL on two separate occasions meets the ADA diagnostic criteria for diabetes. 9)In type 1 diabetes, the C peptide level should be absent, normal, or elevated? Answer: Absent. 10)Why is intravenous triiodothyronine (T3) rarely administered? Answer: It can precipitate dysrhythmias and sudden death. 11) Which hereditary syndromes are associated with pheochromocytomas? Answer: von Hippel-Lindau (VHL) syndrome, multiple endocrine neoplasia type 2 (MEN 2), and neurofibromatosis type 1 (NF1). 12) What is a medical treatment of choice for acromegaly? Answer: Octreotide, a somatostatin analog, 100 mcg subcutaneou 13)Which disease is responsible for over 95% of cases of hypothyroidism? Answer: Primary thyroid disease. 14): What is the most common cause of hyperthyroidism in the United States? Answer: Graves' disease. 15)What is the mechanism of action of dipeptidyl peptidase-4 (DPP-4) inhibitors? Answer: Prolonging active incretin level and thus increasing insulin synthesis and releasing and decreasing hepatic gluconeogenesis. 16)In patients with rhabdomyolysis, what is the classic electrolyte abnormality that must also be addressed? Answer: Hyperkalemia. 17) Which DM1 autoantibodies are the most common? Answer: Those directed against beta-cell glutamic acid decarboxylase. 18)An elevated thyroid peroxidase antibody (TPO) level is diagnostic for which thyroid disorder? Answer: Hashimoto's thyroiditis. 19)A patient is scheduled for a unilateral adrenal tumor resection for suspected adrenocortical carcinoma. Which of the following surgical approaches would be best for this adrenalectomy? open transabdominal approach 20)Which approach is preferred in cases of a bilateral adrenalectomy? Answer: Posterior retroperitoneoscopic.

facts-2

1)What is Kallman syndrome? Answer: Anosmia in addition to hypogonadotropic hypogonadism due to congenitally disrupted migration of gonadotropin-releasing hormone-secreting neurons and olfactory bulbs. 2)In addition to the TSH level, what additional testing should be done in the workup of hyperthyroidism? Answer: Free T4, T3, and thyroid antibodies. 3)What laboratory test can aid in the diagnosis of menopause in a woman who is status post hysterectomy? Answer: Follicle-stimulating hormone (usually > 70 IU/L). 4) What are the most common precipitating events for hyperglycemic hyperosmolar state? Answer: Infection and inadequate insulin treatment/noncompliance. 5)What is the mechanism of action of the thiazolidinediones, pioglitazone and rosiglitazone? Answer: Increase insulin sensitivity in muscle and fat. 6) What is the most common cause of Cushing syndrome? Answer: Exogenous steroid administration. 7) What is the term for deep and labored breathing seen in patients with diabetic ketoacidosis? Answer: Kussmaul respirations. 8)What annual preventative exams are recommended for patients with diabetes? Answer: An annual podiatry exam and an annual eye exam. 9)In addition to the TSH level, what additional testing should be done in the workup of hyperthyroidism? Answer: A free T4 level is also recommended as part of the initial evaluation for hyperthyroidism. Free T4 is elevated in most patients with hyperthyroidism. 10)Which drug class is known to frequently interfere with 24-hour urine fractionated metanephrines and catecholamines? Answer: Tricyclic antidepressants. 11)What are the major adverse effects of sulfonylureas? Answer: Hypoglycemia and weight gain. 12)What is the DM2 screening criteria the US Preventive Services Task Force uses? Answer: Per the USPSTF, screening is recommended in adults 35-70 years of age who are overweight or obese. 13)What diagnostic test would confirm Hashimoto (autoimmune) thyroditis? Answer: Thyroid peroxidase or thyroglobulin antibodies. 14)What symptoms would you expect to find in a patient who is experiencing thyroid storm? Answer: Palpitations, tachycardia, hypotension, dysrhythmia, high fever, nausea, vomiting, psychosis, and agitation. 15)Which of the following is consistent with adrenal insufficiency? Hyponatremia and hyperkalemia 16)Which of the following classes of diabetic medications works mainly by suppressing hepatic gluconeogenesis? 17) A thiazolidinedione should not be used in combination with what other diabetes drug? Answer: Insulin, due to a potentiation of weight gain and fluid retention. 18) What is the most specific test in the diagnosis of primary adrenal insufficiency? Answer: Adrenocorticotropic hormone stimulation test. 19) What are three common forms of diabetic neuropathy? Answer: Autonomic (gastroparesis, orthostasis, constipation), acute mononeuropathy (CN palsy, foot drop) and distal, symmetric, sensory > motor peripheral neuropathy. 20) Which cardiovascular adverse effect can occur with high doses of cabergoline? Answer: Valvular heart disease.

facts-3

According to the American Diabetes Association (ADA) glycemic control goals, which of the following scenarios would prompt a change in insulin dosing in a patient with type 1 diabetes mellitus? AA 10-year-old boy whose fasting serum glucose is 130 mg/dL BA 14-year-old girl whose hemoglobin-A1c is 6.8% CA 4-year-old girl whose hemoglobin-A1c is 7.8% DAn 8-year-old boy whose hemoglobin-A1c is 5.9% An 8-year-old girl presents to your office with complaints of increased thirst, increased volume of urine and weight loss. Physical exam is normal except for the patient appearing slightly ill and with a weight loss of seven pounds since her last well child exam. Hemoglobin A1C is 9.2%. Which of the following is the most appropriate therapy? 8-year-old boy presents with a complaint of excessive thirst and urination. His mother comments that he sleeps "all the time". Which of the following test result scenarios is needed to make a diagnosis of type 1 diabetes mellitus?

A 4-year-old girl whose hemoglobin-A1c is 7.8% The statement includes a new pediatric glycemic control target of HbA1c of less than 7.5% (58 mmol/mol)across all pediatric age groups target A1c level is < 7.5% Which of the following is the most appropriate therapy?= Insulin Two-hour serum glucose >200 mg/dL, checked after a 75g oral glucose tolerance test or fasting serum glucose >126 mg/dL, Diabetes Mellitus Type 1 Patient will be a child Polydipsia, polyphagia, polyuria, and weight loss Diagnosis is made by ADA diagnostic criteriaSymptoms plus one of the following Random plasma glucose ≥ 200 mg/dL Fasting plasma glucose ≥ 126 mg/dL Glycated hemoglobin (A1C) ≥ 6.5% Plasma glucose ≥ 200 mg/dL 2 hours after a 75 g glucose load during an oral glucose tolerance test Most commonly caused by autoimmune destruction of pancreatic cells Treatment is insulin What does hemoglobin A1c represent? Answer: Although the A1C reflects mean blood glucose over the entire 120-day life span of the red blood cell, it correlates best with mean blood glucose over the previous 8 to 12 weeks.

hyrotropin receptor Graves disease, an autoimmune condition in which autoantibodies to the thyrotropin receptor cause increased thyroid hormone synthesis and secretion and diffuse thyroid growth What is the most significant side effect of methimazole? Answer: Teratogenicity.

A 40-year-old woman with a history of hypertension and anxiety presents to the clinic. Her medications include lisinopril and escitalopram. During her visit, she reports worsening anxiety for the past 2 months. She states her escitalopram dose, which previously controlled her anxiety symptoms, no longer seems effective. When describing her anxiety symptoms, she reports new palpitations, bilateral hand tremor, heat intolerance, and diarrhea. She also reports weight loss despite increased appetite. Vital signs are obtained, which reveal HR of 115 bpm, BP of 120/80 mm Hg, RR of 14 bpm, and SpO2 of 98% on room air. Physical exam reveals a thin woman in no acute distress. Periorbital edema, lid retraction, and exophthalmos are present on ophthalmic exam. There is a palpable goiter. Cardiac exam reveals regular rhythm with tachycardia, normal S1 and S2, and no murmurs, rubs, or gallops. Muscle strength is normal in all extremities, but a mild essential tremor is noted in her hands. Her bilateral lower extremities are shown above. The remainder of the physical exam findings are normal. Based on her presentation and physical exam findings, antibodies directed against which of the following is most specific for the presumed diagnosis?

A 23-year-old man with a history of type 1 diabetes presents to the emergency department with abdominal pain, vomiting, and fatigue. He has not been taking his insulin. He is found to have diabetic ketoacidosis. His vital signs and mental status are normal except for RR of 26/min and HR of 112 bpm. He has a normal electrocardiogram. Treatment is initiated with intravenous normal saline and insulin. After 1 hour of insulin therapy, his pH is 7.21, glucose is 190 mg/dL, anion gap is 22 mEq/L, bicarbonate is 13 mmol/L, and potassium is 5.5 mEq/L. Which of the following is the most appropriate next step?

Add dextrose to the intravenous fluids dx: Diabetic ketoacidosis hypovolemia, hyperglycemia, acidemia, and hyperkalemia How is the anion gap calculated? Answer: Anion Gap = Sodium - (Chloride + Bicarbonate).

A 47-year-old woman with a history of hypertension presents to the ED with a severe headache that began suddenly three hours prior to arrival. She has a history of recurrent frontal headaches for the last month and is currently being treated for migraine. Today, she also complains of blurry vision, diplopia, nausea, vomiting, and confusion. Her vital signs are BP 95/45 mm Hg, HR 118 bpm, RR 23/min, T 36.7°C, and pulse oximetry of 98% on room air. Neurologic exam reveals a dilated and minimally reactive left pupil, a globe deviated inferiorly, and bitemporal hemianopsia. Lab results reveal only hyponatremia of 129 mEq/L. Normal saline is administered. A CT scan reveals an intrasellar mass. Which of the following is an appropriate step in the management of this patient? Which of the following is the recommended treatment for a pituitary adenoma >1 cm in size?

Administer intravenous hydrocortisone dx: pituitary apoplexy from a pituitary tumor The most common symptom, in more than 95% of cases, is a sudden-onset headache located behind the eyes or around the temples. It is often associated with nausea and vomiting. The patient's clinical presentation is also consistent with acute (secondary) adrenal insufficiency due to inadequate ACTH production from the pituitary gland. Adrenal insufficiency manifests in this patient as hypotension, fatigue, abdominal pain, and hyponatremia. It is also associated with hyperkalemia and hypoglycemia. Hydrocortisone is the preferred steroid to administer because it provides both glucocorticoid and mineralocorticoid effects. Clinical improvement is usually seen within a few hours of steroid administration. Surgical resection is the treatment of choice for pituitary adenomas >1 cm

A 62-year-old woman with a medical history of hypertension and atrial fibrillation presents to the clinic. She reports a new onset of hand tremors, palpitations, anxiety, and weight loss of 7 pounds in the past 3 weeks. On exam, she is afebrile and tachycardic to 105 bpm and has blood pressure of 120/80 mm Hg and SpO2 of 98% on room air. She appears restless while seated for her exam. Her skin is warm and diaphoretic. Her thyroid is diffusely enlarged, but no nodules are palpated. She has a bilateral hand tremor. The remainder of her physical exam findings are normal. Laboratory evaluation reveals a thyroid-stimulating hormone of 0.15 µU/mL, free triiodothyronine of 1.2 ng/dL, and free thyroxine of 3.4 ng/dL. Which of the following medications is most likely to be the cause of her symptoms based on the above findings? AAmiodarone BDigoxin CDiltiazem DMetoprolol ENivolumab

Amiodarone dx: hyperthyroidism secondary to amiodarone treatment, also known as amiodarone-induced thyrotoxicosis (AIT). Diagnosis is made based on laboratory findings. All patients with primary hyperthyroidism, including AIT, will have low levels of thyroid-stimulating hormone and high levels of triiodothyronine and thyroxine. Treatment of AIT includes oral glucocorticoids for a course of several months What are other serious side effects of amiodarone? Answer: Bradycardia, hypotension, hepatotoxicity (drug-induced phospholipidosis, hepatic cirrhosis), and pulmonary toxicity (acute respiratory distress syndrome, pneumonitis, pulmonary alveolar hemorrhage).

A woman complains of acute onset and rapidly progressive dark hair growth on her upper lip. She has also noticed a deepening of her voice, an increase in muscle bulk and a cessation of normal menses. Abdominal CT scanning reveals a large adrenal gland mass. Histopathologic evaluation shows the presence of malignant cells. An excess of which of the following class of hormones is most likely to be found in this patient?

Androgens Hirsutism What is congenital adrenal hyperplasia? Answer: An autosomal recessive disorder that causes a defect in the 21-hydroxylase enzyme causing defective glucocorticoid biosynthesis and adrenal gland androgen overproduction. ------------------------------------------------------- Mineralocorticoid (D)excess (hyperaldosteronism) causes hypertension, headaches and polydipsia. catecholamines (B), epinephrine and norepinephrine, can result from a pheochromocytoma. This adrenal medulla neoplasia causes hypertension, headache, palpitations, pallor and diaphoresis, not hirsutism and female masculinization. Eicosanoid hormones (C) include prostaglandins, leukotrienes and thromboxanes. These hormones are involved in the immunity and inflammation pathways, not the masculinization pathway

A patient presents with weight loss, nervousness, and palpitations. During physical examination, which of the following signs suggests hyperthyroidism? AAnterior neck bruit BCoarse, dry, scaling skin CDistal muscle weakness DPeriorbital erythema

Anterior neck bruit If the gland is enlarged, consider thyroiditis or hyperthyroidism. Auscultation of an enlarged gland (goiter) may reveal a bruit or soft rushing sound if the patient is in a hypermetabolic state, indicating increased blood flow. Goiter and gland tenderness suggests a hyperthyroid state.

A 35-year-old woman presents for several days of anterior neck pain radiating to her ears, accompanied by dysphagia and "restlessness." She endorses a low-grade fever and fatigue that is "lingering" from her recent flu-like illness. Physical exam shows a tender, symmetrically enlarged thyroid. Laboratory studies show a normal complete blood count, suppressed thyroid stimulating hormone, low antithyroid antibody titers, and a high erythrocyte sedimentation rate. In addition to propranolol, the most appropriate treatment plan includes which of the following medications?

Aspirin dx: subacute thyroiditis Subacute (de Quervain) thyroiditis Preceded by viral URI Anterior neck pain Suppressed TSH, elevated ESR Low uptake on thyroid scan Tx: high-dose ASA or NSAID

A 40-year-old woman with a history of Hashimoto thyroiditis on levothyroxine presents to the clinic with 6 months of nonspecific symptoms, including fatigue, malaise, and weight loss. Vital signs include a heart rate of 80 bpm, blood pressure of 100/60 mm Hg, respiratory rate of 20 breaths per minute, oxygen saturation of 98% on room air, and temperature of 98.6°F. Physical examination reveals brown hyperpigmentation of her skin, particularly on the face, neck, elbows, knees, and waist. Laboratory studies include a thyroid-stimulating hormone of 2 mIU/L, early morning cortisol of 2 mcg/dL, sodium of 127 mEq/L, and potassium of 5.4 mEq/L. Which of the following is the most common cause of the suspected condition? A 44-year-old woman presents to the office with worsening fatigue and dizziness over the past few months. She decided to make an appointment because she had a few near fainting episodes at her indoor office job. After reviewing her chart, you note that she has lost about 15 pounds over the past year. Upon physical exam, you note that she has a blood pressure of 86/62 mm Hg and that she has areas of hyperpigmentation to her face, arms, hands and mucous membranes. Which of the following is most appropriate to diagnose her condition? AAbdominal CT scan BAutoantibody test CProlactin level DRapid adrenocorticotropic hormone (ACTH) stimulation test

Autoimmune adrenalitis Rapid adrenocorticotropic hormone (ACTH) stimulation test Primary Adrenal Insufficiency (Addison Disease) Patient presents with abdominal pain, nausea, vomiting, diarrhea, fever, and confusion PE will show hyperpigmentation of skin and mucus membranes and hypotension Labs will show hyponatremia and hyperkalemia Most commonly caused by autoimmune destruction of the adrenal cortex Treatment is hydrocortisone or other glucocorticoid What medications are indicated in the day-to-day management of primary adrenal insufficiency? Answer: Hydrocortisone (two-thirds of the total daily dose in the morning and one-third of the total daily dose in the evening) and fludrocortisone. What electrolyte abnormalities are typically seen in primary adrenal insufficiency? Hyponatremia and hyperkalemia What is the best diagnostic test for adrenal insufficiency? Answer: Serum cortisol level. What change in glucocorticoid replacement is recommended during minor illnesses in patients with primary adrenal insufficiency? Answer: Three times the usual maintenance dose for 3 days.

A 40-year-old woman presents to the office with complaints of worsening headaches and double vision over the past three months. After reviewing her chart, you notice that she has gained over 40 pounds since her last visit to the office one year ago. Upon physical exam, you note her weight distribution is mainly in the waist, she has increased adipose tissue in the face, proximal muscle weakness and purple striae over the abdomen. She is not currently taking any prescription or over-the-counter medications. What is the most likely cause of her condition?

Benign pituitary adenoma Cushing Syndrome Patient presents with amenorrhea, central obesity, depressive symptoms, and easy bruising PE will show purple striae, moon face (facial adiposity), buffalo hump (increased adipose tissue in the neck and upper back), and hypertension Diagnosis is made by 24-hour urine cortisol and testing ACTH levels Most noniatrogenic cause is hypercortisolism from ACTH-secreting pituitary tumor If cause is pituitary tumor then it's called Cushing disease What is the most common cause of Cushing syndrome? Answer: Exogenous steroid administration. What life-threatening condition results from discontinuing therapy in a patient who has been on long-term corticosteroid treatment? Answer: Adrenal crisis.

A 35-year-old woman presents to her primary care clinician for palpitations for the past 2 months. She states she feels her heart racing, which is often accompanied by sweating, tremors, and anxiety. She also reports 10 pounds of unintentional weight loss over the past 2 months and menstrual irregularities. She has a history of celiac disease and takes no medications at this time. She reports no history of anxiety or depression, chest pain, or shortness of breath. Her vitals include BP of 142/85 mm Hg, HR of 145 bpm, RR of 14/min, T of 99.0°F, and SpO2 of 99% on room air. Physical exam reveals a diffusely enlarged, nontender thyroid, exophthalmos, and warm, moist skin. Her labs are significant for a thyroid-stimulating hormone of 0.2 mU/L, T3 of 400 ng/dL, and a free T4 of 3.5 ng/dL. ECG shows an irregularly irregular rhythm, and chest radiographs reveal no abnormalities. What is the best next step in the management of this patient's condition? ABlock sympathetic hyperactivity BElectrical cardioversion CInhibit the release of stored thyroid hormone DInhibit thyroid hormone synthesis ERadiofrequency ablation

Block sympathetic hyperactivity dx: Graves disease Autoimmune Hyperthyroidism Goiter Exophthalmos Pretibial edema Medications, radioactive iodine, surgery TSH receptor antibodies and antithyroperoxidase are often elevated. Management of this condition starts with blocking sympathetic hyperactivity to help control symptoms and heart rate. Propranolol, a beta-blocker, is the initial treatment of choice for patients with hyperthyroidism. Beta-blockers can help control symptoms of hyperthyroidism by blocking beta-adrenergic receptors in the heart and should be started in patients with hyperthyroidism as soon as possible unless they have a contraindication.

A 32-year-old woman presents to family practice with weight loss and fatigue for the past few months. She has lost approximately 15 pounds in 3 months without trying. She also reports heart palpitations, excessive sweating, and oligomenorrhea. She reports no shortness of breath, illicit drug use, or night sweats. Her medical history includes anxiety, and she takes sertraline 50 mg daily. She is on no other medications. Her vitals include a BP of 128/82 mm Hg, HR of 101 bpm, RR of 12/min, T of 98.7°F, and SpO2 of 100% on room air. She has warm, moist skin, and examination of her thyroid reveals a palpable 1 cm nodule on the left lobe that is nontender. Lab values include a thyroid-stimulating hormone level of 0.2 mU/L, and radioactive iodine uptake scan shows a discrete area of increased uptake, corresponding to the left lobe nodule noted on exam. Which complication is most likely to develop if this patient is left untreated?

Bone loss dx: Hyperthyroidism Complications of hyperthyroidism include bone loss, hypercalcemia, cardiac dysrhythmias, and thyroid crisis. Thyroid hormone stimulates bone resorption and osteoclast activity. This results in a loss of cortical bone density and trabecular bone volume. In a patient with primary hypothyroidism, would the thyroid-stimulating hormone (TSH) level be elevated or depressed? Answer: Elevated. ---------------------------------------------------- Myxedema coma (E) is a serious and life-threatening complication of hypothyroidism. Older women are at higher risk for a myxedema crisis, and other risk factors include prolonged cold exposure or a history of stroke or infection. The mortality rate with myxedema coma is high, and treatment includes rewarming, intravenous liothyronine, and mechanical ventilation.

An otherwise healthy 36-year-old man reports 6 months of decreased libido and inability to maintain erections. He still experiences spontaneous nighttime erections. Vital signs include BP 130/84 mm Hg, HR 65 bpm, RR 15/min, and T 98.6° F. His physical exam is normal, and laboratory testing reveals a low total testosterone level on two separate mornings. Which of the following is the most appropriate next step?

Checking luteinizing hormone and follicle-stimulating hormone Male Hypogonadism Primary: testes dysfunction, elevated FSH and LH Secondary: hypothalamus or pituitary, normal to low FSH and LH Low sperm count Infertility Dx: low morning testosterone on two occasions Tx: testosterone replacement therapy, surgery for tumor What are some of the potential side effects of testosterone therapy? Answer: Polycythemia, blood clots (rare), acne, breast enlargement, and aggressive behavior (more common in boys and teens than adult men)

A 62-year-old man with a history of diabetes mellitus type 2 and hyperlipidemia presents to his family medicine clinic to discuss his most recent blood work. His vital signs are HR of 80 bpm, RR of 18/min, T of 98.6°F, and BP of 130/84 mm Hg. His cholesterol is well controlled on atorvastatin. His total cholesterol is 180 mg/dL, HDL is 60 mg/dL, and LDL is 70 mg/dL. However, he has suboptimal control of his sugars while on metformin, with an A1C at 8.9%. Based on his most recent labs, his primary care clinician discussed adding pioglitazone to his regimen. Which of the following black box warnings should this patient be educated about?

Congestive heart failure side effects include edema, weight gain, fluid retention, new onset or worsening of macular edema, anemia, and an increase in fracture risk in women. There is also a possibility of increase in bladder cancer, specifically with pioglitazone. The single black box warning for pioglitazone is congestive heart failure.

A 13-year-old girl presents to the clinic with her parents who report small stature and no signs of puberty. She takes no medications, has no prior illnesses, and is up-to-date on her vaccinations. Family history reveals her father is 72 in tall and her mother is 64 in tall and had menarche at age 16. The patient's vital signs are within normal limits, her body mass index is 15 kg/m2, and her height is 55 in. Physical exam reveals a healthy looking girl with no breast buds or pubic hair. Several laboratory results are listed below. Thyroid-stimulating hormone: 1.5 mIU/L T4: 80 ng/dL Luteinizing hormone: 3 IU/L Follicle-stimulating hormone: 4 IU/L Hemoglobin A1C: 5.0% Prolactin: 2 ng/ml Estradiol: 20 pmol/L Growth hormone: 12 nmol/L Which of the following is the most likely diagnosis?

Constitutional delay of puberty Puberty is considered delayed when there are no signs of breast development by age 13 in a girl or no signs of testicular enlargement in a boy by age 14. Constitutional delay of puberty refers to puberty that is delayed in the absence of other pathology and that subsequently and spontaneously resumes progress at a later age.

You are treating a patient with type 2 diabetes mellitus. He is currently on metformin, but his hemoglobin-A1c is not at goal. You decide to add insulin. Which of the following is the best approach when considering this management plan?

Continue oral medications, add long-acting insulin dx: Diabetes Mellitus Type 2 Screen adults aged 35-70 with BMI ≥ 25 kg/m2 every 3 yearsThose with additional risk factors need annual screening

Which of the following is most likely to be seen on the physical exam of a patient with hypothyroidism?

Delayed relaxation of deep tendon reflexes Hypothyroidism Patient presents with generalized weakness, fatigue, facial swelling, constipation, cold intolerance, and weight gain PE will show periorbital edema, dry skin, and coarse brittle hair Labs will show high TSH and low free T4, antithyroid peroxidase and antithyroglobulin autoantibodies Most commonly caused by Hashimoto thyroiditis Treatment is levothyroxine Takes about 6 weeks to see treatment effectsMonitor TSH Hashimoto: risk factor for non-Hodgkin lymphoma True or false: carpal tunnel syndrome is a common complication of hypothyroidism? Answer: True.

A 64-year-old man presents to the clinic for management of chronic symptomatic hypercalcemia of malignancy. Despite the continued use of bisphosphonates, his total calcium level remains elevated. Which of the following is the most appropriate medication this patient should receive next?

Denosumab a monoclonal antibody which inhibits bone resorption via inhibition of receptor activator of nuclear factor kappa-B ligand (RANKL) can be used. In addition, denosumab is an option for patients in whom bisphosphonates are contraindicated due to severe renal impairment, as it is not renally excreted. What is the most common electrocardiogram manifestation of severe hypercalcemia? Answer: Severe hypercalcemia can be associated with shortening of the QT interval on electrocardiogram.

A patient presents with excessive urination and thirst. A comprehensive metabolic panel is normal. Urinalysis is significant for a low specific gravity. Which of the following is the most appropriate next test to order for the evaluation of these symptoms?

Desmopressin stimulation test What is urine specific gravity? Answer: A measure of urine solute concentration. High values indicate a concentrated urine, Low values indicate a dilute urine. Diabetes Insipidus (DI) Patient presents with polyuria and polydipsia Labs will show increase in plasma osmolality and a decrease in urine osmolality Central DI: most commonly caused by a decrease in ADH productionDiagnosis is made by vasopressin challenge test: > 50% increase in urine osmolality and decreased urine volumeTreatment is intranasal DDAVP to reduce nocturia Nephrogenic DI: history of taking lithiumMost commonly caused by kidney unresponsiveness to ADHDiagnosis is made by water deprivation test: no change in urine osmolalityTreatment is HCTZ, amiloride, indomethacin

A 22-year-old man presents to the emergency department with 5 days of worsening abdominal pain and vomiting. He has had no appetite since his symptoms started and has lost about 20 pounds in the past 2 weeks. His vital signs are a HR of 125 bpm, RR of 19/min, T of 98.8°F, and BP of 100/62 mm Hg. On physical exam, the patient has a rapid heart rate and rhythm; breath sounds are clear to auscultation bilaterally; there is hyperpigmentation to his skin over the knuckles, elbows, and knees; and his abdomen is diffusely tender throughout, but he has no guarding or rebound. His basic metabolic panel shows hyponatremia and hyperkalemia. His glucose level is at 59 mg/dL. The patient was given 2 L of saline bolus. What should be administered immediately after an initial bolus of fluids? ADexamethasone BDopamine CKetoconazole DLevofloxacin ELevothyroxine

Dexamethasone dx: acute adrenal crisis. This is an acute emergency,

The clinic receives a report that one of your patients has congenital hypothyroidism that was identified by newborn screening. You review the chart and the patient was last seen at around two weeks of age. At that time, the infant had regained birth weight and was breastfeeding well, and the mother did not have any concerns. Your physical examination at that time was normal. You call the mother, and she reports that the infant is doing well. Which of the following is the next best step? ADraw thyroid stimulating hormone (TSH) level BRefer to a pediatric endocrinologist CRepeat newborn screening DStart levothyroxine

Draw thyroid stimulating hormone (TSH) level

A 64-year-old man with a history of diabetes mellitus on metformin and a previous non-ST elevation myocardial infarction presents to the clinic to follow up on his diabetes management. Vital signs today include a heart rate of 80 bpm, blood pressure of 120/80 mm Hg, respiratory rate of 20/minute, oxygen saturation of 98% on room air, and a temperature of 98.6°F. Physical examination reveals a regular cardiac rate and rhythm and lungs that are clear to auscultation. The patient's laboratory findings include a creatinine of 1.2 mg/dL and a hemoglobin A1C of 7.7%. Which of the following medications would decrease cardiovascular morbidity and mortality in this patient? AEmpagliflozin BGlimepiride CGlipizide DLinagliptin ERepaglinide

Empagliflozin Sodium-glucose cotransporter 2 (SGLT2) inhibitors, such as empagliflozin and canagliflozin, are a class of medication used to lower blood glucose in patients with diabetes mellitus. This medication class works by blocking the reabsorption of glucose in the proximal tubules of the kidneys, which leads to increased renal excretion of glucose. What are the most common adverse effects of metformin? Answer: Gastrointestinal (nausea, vomiting, diarrhea, and abdominal discomfort) symptoms and a metallic taste in the mouth.

A 62-year-old man is admitted to the hospital for sepsis secondary to a urinary tract infection. His medical history is significant only for hypertension. On examination he has a temperature of 36.5°C, a TSH level of 0.2 mcIU/mL, and a T3 level of 60 ng/dL. Which one of the following is the most likely explanation for these findings?

Euthyroid sick syndrome abnormal findings on thyroid function tests that occur in the setting of a nonthyroidal illness, without preexisting hypothalamic-pituitary and thyroid gland dysfunction. This is most commonly diagnosed in a hospitalized patient and after recovery from the underlying illness. An elevated thyroid peroxidase antibody (TPO) level is diagnostic for which thyroid disorder? Answer: Hashimoto's thyroiditis.

A 19-year-old woman with no significant medical history presents to her primary care office with concerns of being overweight. She has been trying to lose weight with a strict diet and exercise plan for over 4 months, but she has been unsuccessful. Her family history consists of her mother who has hypothyroidism and her father who has hypertension. Her vital signs are a HR of 105 bpm, RR of 18/min, BP of 120/77 mm Hg, T of 97.8°F, and BMI of 21.6 kg/m2. On physical exam, her heart is normal, she has no murmurs, and no palpable thyroid nodules are identified. She recently had annual labs completed, which show low thyroid-stimulating hormone, high T4, and undetectable thyroglobulin levels. Which of the following is the most likely diagnosis for this patient?

Exogenous intake of thyroid hormone Labs should be consistent with elevated T4, low TSH, and undetectable thyroglobulin levels. In addition, a 24-hour radioiodine uptake can be done, which would show either low or nearly absent radioiodine uptake values. Treatment would consist of discontinuing the thyroid hormone. ------------------------------------------------------------------ Graves disease would also have positive thyroid-stimulating immunoglobulin levels. On exam, they would have an enlarged thyroid, exophthalmos or proptosis, and sometimes pretibial myxedema. A radioactive uptake scan for Graves disease would show diffuse radioiodine uptake.

A 32-year-old woman with a body mass index of 35.1 kg/m2 presents to your office with increased thirst, unintentional weight loss, and blurry vision over the past month. Which of the following is the most appropriate diagnostic test?

Fasting plasma glucose What are the indications used to determine which asymptomatic adults need to be screened for diabetes? Answer: Blood pressure > 135/80 mm Hg, age ≥ 45 years, or overweight plus one additional risk factor, such as family history of diabetes in a first-degree relative, elevated blood pressure, or elevated cholesterol. Diabetes Mellitus Type 2 Hyperglycemia due to insulin resistance and progressive loss of insulin secretion Risk factors: obesity, abdominal adiposity, family history, lifestyle factors Sx: asymptomatic, polydipsia, polyphagia, polyuria ADA diagnostic criteria:SymptomaticRandom plasma glucose ≥ 200 mg/dLAsymptomaticFasting plasma glucose ≥ 126 mg/dL Glycated hemoglobin (A1C) ≥ 6.5%Plasma glucose ≥ 200 mg/dL 2 hours after a 75 g glucose load during an OGTT Tx: lifestyle modifications then medication (first-line Rx: metformin) Screen adults aged 35-70 with BMI ≥ 25 kg/m2 every 3 yearsThose with additional risk factors need annual screening

60-year-old woman with a history of type 2 diabetes mellitus on metformin presents to the clinic to follow up a lump palpated on her thyroid during a previous routine well examination. The patient reports no dysphagia or voice changes. Vital signs today include a heart rate of 75 bpm, blood pressure of 120/80 mm Hg, respiratory rate of 20/min, oxygen saturation of 99% on room air, and temperature of 98.6°F. Her thyroid-stimulating hormone level is 2 mU/L, and a thyroid ultrasound reveals a 1.3 cm hypoechoic nodule with irregular borders and microcalcifications. What is the best next step in the management of this patient's condition?

Fine-needle aspiration biopsy The initial workup includes a thyroid-stimulating hormone (TSH) level and thyroid ultrasound. The next step varies based on the sonographic appearance of the thyroid nodule. Sonographic features suspicious for malignancy include size > 1 cm, irregular margins, microcalcifications, taller than wider shape, evidence of extrathyroidal expansion, and hypoechoic appearance. Fine-needle aspiration (FNA) biopsy is the recommended next step for patients with sonographic features concerning for malignancy. What is the most common type of thyroid cancer? Answer: Papillary.

A 19-year-old man presents after an intentional ingestion of an unknown substance. He complains of visual disturbances. He has an elevated anion gap acidosis and an elevated osmolar gap. Vital signs are within normal limits. Serum ethanol, lactic acid, and salicylate levels are normal. Which of the following is the correct antidotal therapy?

Fomepizole (and ethylene glycol) poisoning. It inhibits alcohol dehydrogenase, slowing the conversion of methanol into toxic metabolites. The unmetabolized methanol is then renally excreted. Other therapies include sodium bicarbonate, folinic acid and hemodialysis. dx: Methanol is a compound found in antifreeze, windshield washer fluid, cleaning agents, formalin, and embalming fluid. It is also a byproduct of illicit alcohol production It causes a metabolic acidosis by inhibiting mitochondrial cytochrome oxidase. An osmolar gap (the difference between the calculated and measured serum osmolarity) is caused by elevated levels of methanol in the serum. Autopsy studies suggest that retinal cells have unique enzymes which convert formic acid to formate, an especially toxic compound. Therefore, visual impairment Methanol Intoxication Antifreeze, windshield washer fluid, paint thinners Methanol → formic acid (toxic metabolite) Visual Sx (snowstorm) Anion gap metabolic acidosis, increased osmolar gap Rx: fomepizole (first line), folinic acid, bicarbonate

A 64-year-old man presents to the clinic for a routine complete physical examination. His history is significant for type 2 diabetes mellitus, hypertension, and obesity. His current medications include amlodipine, aspirin, furosemide, metformin, and saxagliptin. His blood glucose logs show poorly controlled diabetes, and a recent hemoglobin A1C is 8.9%. If empagliflozin is added to his medication regimen to increase glycemic control, which of the following medications will need to be adjusted? What class of antihypertensive medications can negate the antidiabetic effects of empagliflozin?

Furosemide Sodium-glucose cotransporter 2 (SGLT-2) inhibitor medications inhibit the reabsorption of glucose and lower the threshold of urinary glucose excretion from 180 mg/dL to around 40 mg/dL. These mechanisms allow serum glucose reduction through enhanced urinary glucose excretion. Loop diuretics such as furosemide reduce sodium reabsorption, thereby increasing fluid excretion. What class of antihypertensive medications can negate the antidiabetic effects of empagliflozin? Answer: Thiazide diuretics. -------------------------------------------------------- Metformin (D) reduces hepatic gluconeogenesis and lipogenesis, thus lowering serum glucose levels. Metformin and empagliflozin are safe to take together and are often combined for better glucose control. Saxagliptin (E) is a dipeptidyl peptidase 4 inhibitor that enhances glucose-dependent insulin secretion and slows gastric emptying. This medication does not interact with empagliflozin.

A 45-year-old woman with a history of type 2 diabetes mellitus presents to your office with a complaint of occasional episodes of diaphoresis, anxiety, tremors, and weakness. Which of the following medications is most likely the cause of her symptoms?

Glipizide dx: Hypoglycemia Hypoglycemia Usually glucose < 60 mg/dL Confusion, agitation, unresponsiveness Tachycardia, diaphoresis, tremulousness Focal neurologic deficit Dextrose, thiamine, glucagon At what self-monitored glucose level should patients be concerned about hypoglycemia? Answer: ≤ 70 mg/dL.

A 57-year-old type 2 diabetic man presents to his primary care provider for his annual physical. He was diagnosed with diabetes seven years ago and has a history of alcohol use disorder. His most recent hemoglobin A1C was 7.9 percent and fasting glucose of 156. He is not on any diabetic medications. He states that two years ago he had an episode of intense mid-abdominal pain that radiated to his back. He has had two more similar episodes. Ever since then, he has constant epigastric pain and has noticed loose, foul smelling stools that float in the toilet. Considering his glycemic control and past medical history, what class of anti-hyperglycemic agents is contraindicated for this patient?**** ABiguanides BGlucagon-like peptide-1 agonist CSulfonylureas DThiazolidinediones

Glucagon-like peptide-1 agonist Glucagon-like peptide-1 agonists (GLP-1) are contraindicated in patients with a history of pancreatitis What animal was exenatide, a GLP-1 receptor agonist, first isolated from? Answer: Exenatide was first isolated from the saliva of the Gila monster, a venomous lizard.

A 43-year-old woman presents with palpitations. Vital signs are BP 135/90 mm Hg, HR 115 bpm, and RR 16/min. She is diaphoretic and has a fine tremor. Her eyelids are retracted, and it appears as if she is staring at you. Initial laboratory evaluation reveals a decreased thyroid-stimulating hormone level and elevated antithyrotropin receptor antibodies. Which of the following is the most likely etiology of her symptoms?

Graves disease Hyperthyroidism Sx: heat intolerance, palpitations, weight loss, tachycardia, and anxiety PE: hyperreflexia, goiter, exophthalmos, pretibial edema Labs: low TSH and high free T4 Most commonly caused by Graves disease (autoimmune against TSH receptor) Tx: methimazole or PTU PTU in the first trimester of pregnancy Signs and symptoms manifest as an accelerated body function picture: nervousness, irritability, restlessness, tremor, diaphoresis, tachycardia, dysrhythmias (especially atrial fibrillation), unintentional weight loss (usually with increased appetite), thin skin, brittle hair, poor sleep, digestive hypermobility with diarrhea, polyuria and polydipsia, menstrual irregularities, osteoporosis, muscular weakness(especially proximal weakness), and lower leg nonpitting edema (pretibial myxedema, specific to Graves hyperthyroidism). Laboratory abnormalities include increased free T4, increased free T3, and decreased thyroid-stimulating hormone. What is the difference between proptosis and ptosis? Answer: Ptosis is a drooping upper eyelid commonly due to the muscle weakness associated with myasthenia gravis, a different autoimmune disease (but it can occur with Graves disease).

Which of the following best describes the etiology of the jaundice seen in patients with thyroid storm? ADirect constricting effects of thyroid hormone on the biliary duct BHepatic tissue hypoxia due to increased peripheral consumption of oxygen CHypotension leading to decreased gut motility DImpaired reabsorption of thyroid hormone in the enterohepatic circulation

Hepatic tissue hypoxia due to increased peripheral consumption of oxygen Signs and symptoms of thyroid storm include hyperpyrexia, nausea, vomiting, diarrhea,mental status changes, jaundice, high-output congestive heart failure, cardiac tachyarrhythmias, hypertension,and diaphoresis Why is iodine used in the treatment of thyroid storm? Answer: In pharmacologic doses, iodine inhibits the release of T3 and T4 and inhibits the organification of iodine

A 28-year-old woman presents to the clinic with gradual weight gain for the past 6 months. The patient also reports that her menstrual cycles have been infrequent and heavy. Vital signs today include a heart rate of 80 bpm, blood pressure of 155/95 mm Hg, respiratory rate of 20/minute, oxygen saturation of 98% on room air, and temperature of 98.6°F. Physical examination reveals a regular heart rate and rhythm and lungs that are clear to auscultation. You also notice excess dark hair around the patient's mouth and chin and that the patient has increased abdominal and facial fat with proximal muscle wasting. Which of the following laboratory findings is most likely to be seen? AHigh 24-hour urinary free cortisol level, high adrenocorticotropic hormone, and high dehydroepiandrosterone sulfate BHigh 24-hour urinary free cortisol level, low adrenocorticotropic hormone, and high dehydroepiandrosterone sulfate CHigh 24-hour urinary free cortisol level, low adrenocorticotropic hormone, and low dehydroepiandrosterone sulfate DLow 24-hour urinary free cortisol level, high adrenocorticotropic hormone, and low dehydroepiandrosterone sulfate ELow 24-hour urinary free cortisol level, low adrenocorticotropic hormone, and low dehydroepiandrosterone sulfate

High 24-hour urinary free cortisol level, high adrenocorticotropic hormone, and high dehydroepiandrosterone sulfate Cushing Syndrome Patient presents with amenorrhea, central obesity, depressive symptoms, and easy bruising PE will show purple striae, moon face (facial adiposity), buffalo hump (increased adipose tissue in the neck and upper back), and hypertension Diagnosis is made by 24-hour urine cortisol and testing ACTH levels Most noniatrogenic cause is hypercortisolism from ACTH-secreting pituitary tumor If cause is pituitary tumor then it's called Cushing disease What is the most common cause of Cushing syndrome? Answer: Exogenous steroid administration. What bone abnormality is common in Cushing's syndrome? Answer: Osteoporosis.

A 42-year-old woman presents for evaluation of irregular menses associated with episodes of excessive daytime sweating and palpitations. She has not had a hysterectomy or oophorectomy. She is currently sexually active, although intercourse has been more uncomfortable lately. Which of the following disorders would you consider in this patient's differential diagnosis?

Hyperthyroidism

Which of the following laboratory findings would you expect to find in primary hyperaldosteronism? AHypokalemia BHyponatremia CLow serum aldosterone DMetabolic acidosis

Hypokalemia dx: Hyperaldosteronism Most commonly leads to secondary hypertension. The most common cause is due to a aldosterone-producing adenoma, also referred to as Conn disease. Hypertension and hypokalemia are classically found, Hyperaldosteronism ↑ Na + ↓ K + HTN 1º: adrenal adenoma (most common cause), ↓ renin 2º: ↓ kidney perfusion → RAAS activation, ↑ renin Rx: spironolactone (+ surgery if 1º) or eplerenone (aldosterone antagonist) to normalize potassium levels. Side effects include gynecomastia, rash, impotence, and GI discomfort. Unilateral adrenalectomy is also an option in patients with a single adenoma.

A 40-year-old woman presents with weakness, fatigue, nausea, and diarrhea. Physical exam reveals orthostatic hypotension and axillary fold hyperpigmentation. Which of the following laboratory abnormalities would you expect to find in this patient?

Hyponatremia dx: Primary adrenal insufficiency (Addison disease), or hypocortisolism The most specific sign of primary adrenal insufficiency is hyperpigmentation, typically of the mucous membranes, axillary folds, and nipples.

n 83-year-old woman with a history of hypothyroidism, diabetes mellitus and hypertension is brought to the emergency department with altered mental status. She is found to be hypothermic, hypotensive and bradycardic. Puffiness in the hands and face, a thickened nose and swollen lips are noted on physical exam. Pupils are normal size, equal and reactive to light. Which of the following is the most likely diagnosis? Which of the following is associated with myxedema coma? What electrolyte abnormality is commonly seen in myxedema coma?

Hypotension What electrolyte abnormality is commonly seen in myxedema coma? Answer: Hyponatremia. Why is intravenous triiodothyronine (T3) rarely administered? Answer: It can precipitate dysrhythmias and sudden death. Myxedema Coma Hypothyroidism exacerbation → ↓ metabolic state + AMS PE: stupor, hypoventilation, hypotension, bradycardia Rx: IV thyroid hormone replacement, glucocorticoids High mortality

A 47-year-old woman presents to your office for follow up on her newly diagnosed hypothyroidism. Six weeks ago you prescribed levothyroxine after her thyroid-stimulating hormone was found to be elevated. She has been taking her medication as instructed and her symptoms improved, but retesting of her thyroid-stimulating hormone shows that it is still elevated. Which of the following is the next best step in management? AChange medication to a thyroid hormone preparation containing T3 BDecrease dose of levothyroxine and repeat TSH testing in 6 weeks CIncrease dose of levothyroxine and repeat TSH testing in 6 weeks DMaintain current dose of levothyroxine and repeat TSH testing in 6 weeks

Increase dose of levothyroxine and repeat TSH testing in 6 weeks True or false: pregnant women require more thyroid hormone and may require dosing adjustments to their thyroid replacement during pregnancy? Answer: True.

A 43-year-old woman with type 2 diabetes mellitus presents for a routine visit. She brings home glucose logs that reveal elevated postprandial glucose levels and normal fasting glucose levels. Her current medications include metformin and once-daily long-acting insulin. Her hemoglobin A1C is 8.7%. The decision is made to add an additional three times daily therapy before meals for better glycemic control. Which of the following therapies is most appropriate? AInsulin aspart BInsulin detemir CInsulin glargine DNeutral protamine Hagedorn insulin EU-500 regular insulin

Insulin aspart rapid onset of action and a shorter duration than regular insulin Besides insulin aspart, what are the two other rapidly acting insulin analogs? Answer: Insulin lispro and insulin glulisine.

A 65-year-old man with a medical history of type 2 diabetes, hypertension, and chronic kidney disease stage 2 presents to the emergency department. His current medications include insulin, metformin, lisinopril, and amlodipine. The man is accompanied by his partner who states he has been confused for the last several hours. The man also reports increased sweating, headache, dizziness, and hand tremors. His vital signs are temperature of 37°C, HR of 110 bpm, BP of 120/80 mm Hg, RR of 14 bpm, and SpO2 of 98% on room air. Physical exam reveals a well-appearing man with a BMI of 35 kg/m2 in no acute distress. He is somnolent but answers questions appropriately. He is oriented to person, place, and date. He has a mild resting tremor in both hands. The remainder of his physical exam is unremarkable. Laboratory testing reveals WBC of 4,500/µL, hemoglobin of 15 g/dL, platelets of 200,000/µL, sodium of 137 mEq/L, potassium of 4.0 mEq/L, chloride of 100 mEq/L, bicarbonate of 25 mEq/L, blood urea nitrogen of 30 mg/dL, creatinine of 2.3 mg/dL, and glucose of 40 mg/dL. His thyroid-stimulating hormone level is 3.0 µU/mL. His kidney function compared to labs from his last clinic visit 6 months ago has worsened (creatinine was previously 1.8 mg/dL). Which of the following is the most likely cause of the patient's symptoms?

Insulin-induced hypoglycemia insulin-induced hypoglycemia. Hypoglycemia is defined as plasma glucose concentration ≤ 70 mg/dL. What is the pathogenesis of hypoglycemia in sepsis? Answer: Diminished glycogen stores, impaired gluconeogenesis, and increased peripheral glucose use.

A 43-year-old man presents with altered mental status. His vital signs are HR 113 bpm, BP 143/63 mm Hg, T 98.9°F, and blood glucose of 750 mg/dL. During your evaluation, he has a brief generalized tonic-clonic seizure. Which of the following represents the most important initial intervention for this patient?**** AAlteplase BInsulin bolus and continuous infusion CIntravenous normal saline DPhenytoin

Intravenous normal saline dx:Hyperosmolar Hyperglycemic State What are the most common precipitating events for hyperglycemic hyperosmolar state? Answer: Infection and inadequate insulin treatment/noncompliance.

A 28-year-old man presents to his primary care clinician with concerns for low libido and erectile dysfunction. His medical history includes insulin resistance, hypertension, and carpal tunnel syndrome. He takes metformin 500 mg once daily and lisinopril 10 mg once daily. Vitals include BP of 130/78 mm Hg, HR of 90 bpm, RR of 13/min, T of 98.7°F, SpO2of 99% on room air, height of 6 feet 6 inches, and he weighs 250 pounds. On physical exam, he has large hands with wide fingers and a moist, doughy handshake. His mandible is prominent with coarse facial features. What additional finding is most likely to be associated with this patient's condition? A 55-year old woman presents with feeling "swollen all over." You do not notice any signs of edema, despite her claim that her rings no longer fit and that she has gone from a shoe size of 6 to a 7 1⁄2 over the last 2 years. A review of systems reveals that she is sweating more than usual, feels fatigued, and often has a dull headache and diffuse arthralgias. On examination she has no joint swelling, erythema, tenderness and no skin abnormalities are noted. This presentation is most consistent with which one of the following? Which of the following labs should be ordered first when a patient has suspected acromegaly?

Left ventricular hypertrophy HTN Insulin-like growth factor 1 Acromegaly Patient presents with increased head, glove, or shoe size PE will show coarse facial features, oily skin, visual field deficits, diabetes Labs will show increased IGF-1 Most commonly caused by a pituitary adenoma Treatment is transsphenoidal resection What is the most specific dynamic test for establishing the diagnosis of acromegaly? Answer: Oral glucose tolerance test. What is a medical treatment of choice for acromegaly? Answer: Octreotide, a somatostatin analog, 100 mcg subcutaneou

A patient with rheumatoid arthritis presents with new onset generalized weakness, fatigue, facial swelling and weight gain. Examination is significant for periorbital edema, dry skin and coarse brittle hair. Laboratory testing reveals a high titer of antithyroid antibodies. Which of the following will most likely be prescribed?

Levothyroxine dx:Hashimoto's thyroiditis. List some risk factors for hypothyroidism? Answer: Women over 60 years old, family or personal history of autoimmune disease, prior thyroid surgery or neck radiation. What is the name of the sign that refers to the lateral thinning of the eyebrows as seen in hypothyroidism? Answer: Queen Anne sign. Hypothyroidism Patient presents with generalized weakness, fatigue, facial swelling, constipation, cold intolerance, and weight gain PE will show periorbital edema, dry skin, and coarse brittle hair Labs will show high TSH and low free T4, antithyroid peroxidase and antithyroglobulin autoantibodies Most commonly caused by Hashimoto thyroiditis Treatment is levothyroxine Takes about 6 weeks to see treatment effects Monitor TSH Hashimoto: risk factor for non-Hodgkin lymphoma

A 7-year-old boy presents to the ED with his mother with abdominal pain and vomiting. His mother notes he has been very thirsty over the past several weeks and urinating often. On physical exam, you note diffuse tenderness without rebound or guarding. Vital signs are BP 86/48 mm Hg, HR 140 beats per minute, RR 40 breaths per minute, and T 100.6°F. Which of the following laboratory abnormalities would you expect to find? AHypernatremia BHypocalcemia CLow bicarbonate DLow serum lactate

Low bicarbonate dx: diabetic ketoacidosis (DKA). DKA is more common in insulin-dependent diabetics. The diagnosis of DKA involves several laboratory abnormalities including glucose levels usually > 300 mg/dL, an anion gap metabolic acidosis with low bicarbonate,

A 26-year-old woman who is 16 weeks pregnant and has a history of Graves disease on propylthiouracil presents to the endocrinology clinic for routine follow-up. Vital signs today include a heart rate of 80 bpm, blood pressure of 120/80 mm Hg, respiratory rate of 20/min, oxygen saturation of 98% on room air, and temperature of 98.6°F. Physical examination reveals no lid lag, exophthalmos, tremor, or goiter. Laboratory studies include a thyroid-stimulating hormone level of 2 mU/L and a free thyroxine level of 1 ng/dL. Which complication will be reduced by switching this patient's therapy from propylthiouracil to methimazole? True or false: propylthiouracil is the recommended thionamide for lactating patients with hyperthyroidism.

Maternal hepatotoxicity True or false: propylthiouracil is the recommended thionamide for lactating patients with hyperthyroidism. Answer: False. Methimazole is recommended for patients who are lactating.

A 57-year-old man is evaluated in the clinic for a routine physical exam. He is followed for hypertension and hyperlipidemia. He has no complaints except for some fatigue and constipation. He exercises regularly, eats healthy and takes all of his medications. His medications include atorvastatin 40 mg per day and lisinopril 2.5 mg per day. His blood pressure is 126/80 and body mass index is 23. His physical exam is unremarkable. Laboratory studies reveal elevated total cholesterol and LDL levels. Fasting blood glucose is normal. Which of the following is the most appropriate next step in management?

Measure a thyroid stimulating hormone, what are some common drugs associated with dyslipidemia? Answer: Thiazide diuretics, beta blockers and oral estrogens can all cause modest changes in serum lipid levels.

You are treating a patient with type 2 diabetes mellitus with three different classes of medications. Which of the following medications is proven to decrease mortality in this disease? A 15-year-old girl is seen in your office after being diagnosed with type 2 diabetes mellitus six months ago. She has not done well with diet and exercise, and you decide to add a pharmacologic agent. Which of the following is the correct mechanism of action of the drug of choice? A 67-year-old man is being treated for type 2 diabetes. He has a history of obesity with a BMI of 33 kg/m2 and class I congestive heart failure. Which of the following classes of diabetic medications works mainly by suppressing hepatic gluconeogenesis?

Metformin ( Biguanides) Decreases hepatic glucose production and increases cell insulin sensitivity Which medications used to treat diabetes type 2 has the greatest effect in decreasing hemoglobin-A1c levels? Answer: Metformin, the glinides and the sulfonylureas.

A 60-year-old man with insulin-dependent diabetes mellitus, gastroparesis, hypertension, and congestive heart failure has just received a new diagnosis of pheochromocytoma. Which of the following medications should be discontinued? AFurosemide BInsulin lispro CLisinopril DMetoclopramide

Metoclopramid Metoclopramide blocks the inhibitory effects of dopamine receptor activation on sympathetic nerves, which can potentially lead to catecholamine release in the setting of a pheochromocytoma.

A 38-year-old woman presents with fatigue, anorexia, muscle aches and hyperpigmentation. Laboratory tests are significant for a sodium level of 128 and a potassium level of 5.6. Which of the following is the most appropriate to obtain next in the work up of this patient's condition?

Morning serum cortisol level Primary Adrenal Insufficiency (Addison Disease) Patient presents with abdominal pain, nausea, vomiting, diarrhea, fever, and confusion PE will show hyperpigmentation of skin and mucus membranes and hypotension Labs will show hyponatremia and hyperkalemia Most commonly caused by autoimmune destruction of the adrenal cortex Treatment is hydrocortisone or other glucocorticoid Why is androgen replacement indicated in women but not men with Addison's disease? Answer: Male testes produce adequate testosterone, however the adrenals are the main source of androgen production in women

A 36-year-old woman presents with numbness of her arms and legs for 2 hours. She is status post total thyroidectomy 3 days prior to presentation and is on thyroid hormone replacement therapy. Vital signs are normal and the patient's exam is significant for a clean, well-healing surgical incision on her neck and twitching at the corners of the mouth with tapping at the angle of the jaw. After intravenous repletion of the suspected electrolyte disorder the patient remains persistently low. What is the next step in management? AObtain a non-contrast CT scan of the head BObtain magnesium level CReplete potassium DStop thyroid hormone replacement

Obtain magnesium level dx: hypocalcemia Hypocalcemia Sx: seizures, paresthesias PE: Chvostek sign (facial muscle contraction), Trousseau sign (carpopedal spasm), hyperreflexia Labs: serum Ca2+ < 8.5 mg/dL ECG:↑ QT interval Tx: IV replacement for calcium ≤ 7.5 mg/dL (1.9 mmol/L) or tetany, seizure, carpopedal spasm, prolonged QT Why is it required to administer 3 grams of calcium gluconate and only 1 gram of calcium chloride? Answer: 1 gram of calcium gluconate contains 1/3 of the amount of calcium as 1 gram of calcium chloride due to the difference in the size of the two molecules.

A 54-year-old woman with a history of hyperthyroidism presents to the ED with fever, vomiting, palpitations, and tremors. Which of the following would you expect to find on physical exam? ADry, scaling skin BLoss of eyebrow hair CMaculopapular rash DOcular proptosis

Ocular proptosis ---------------------------------------- Loss of eyebrow hair (B) and dry scaling skin (A) are commonly seen with hypothyroidism. A maculopapular rash (C) is occasionally seen with thyroid disease, however, there is no definitive correlation.

A 19-year-old woman presents to the emergency department with severe flushing, tremors, and vision changes. Vital signs are BP 240/110 mm Hg, HR 104 bpm, RR 20/min, and T 100.4°F (38°C). She was recently found to have a mass on her kidney. Which of the following is the most appropriate course of treatment for this patient? 55-year-old female is awaiting adrenalectomy due to a pheochromocytoma. Prior to surgery, which of the following classes of medications is most likely to be used to control hypertension?

Phentolamine followed by labetalol which of the following classes of medications is most likely to be used to control hypertension? Alpha-adrenergic blockers dx: pheochromocytoma in patients with pheochromocytoma is a two-step process. You must obtain alpha-blockade prior to beta-blockade. This is classically done by administering phenoxybenzamine, which is an alpha-1 and alpha- 2 blocker, or phentolamine, a nonspecific alpha-blocker, followed by a beta blocker, such as labetalol or propranolol. Without appropriate alpha-blockade prior to beta-blockade, unopposed alpha activity can precipitate a malignant hypertensive crisis. Definitive management is by surgical resection of the tumor. Pheochromocytoma Catecholamine-secreting tumor located in the adrenal glands Sx: paroxysmal headaches, diaphoresis, palpitations, tremors, and vision changes PE: hypertension, orthostasis Dx: ↑ 24h urinary catecholamines and metanephrines, or ↑ plasma metanephrine levels, adrenal CT or MRI Tx: alpha-blocker (phenoxybenzamine) prior to beta-blockade to prevent unopposed alpha-agonismsurgical resection Associated with MEN2 (medullary thyroid cancer, pheochromocytoma, +/- primary hyperparathyroidism) What recreational drug mimics the effects of pheochromocytoma? Answer: Cocaine. Which drug class is known to frequently interfere with 24-hour urine fractionated metanephrines and catecholamines? Answer: Tricyclic antidepressants.

A 24-year-old woman presents to the emergency department with a severe headache, palpitations, and tremors after running a 5K race for the first time. She recently lost 20 pounds unintentionally. Vital signs are BP 230/120 mm Hg, HR 100 bpm, RR 24/min, and T 101.3°F (38.5°C). Which of the following is most likely to aid in making the diagnosis? A 45-year-old man presents to the office complaining of a "racing heart", palpitations, increased sweating, and headaches for the past three months. He denies any new life stressors and history of anxiety or panic attacks. He is not currently on any medications. The patient reports his father experienced similar symptoms when he was around the same age and was ultimately diagnosed with a tumor on his adrenal gland. Patient's blood pressure in the office is 164/98 mm Hg and his pulse rate is 88 beats per minute with a regular rhythm. Physical examination reveals a diaphoretic, well developed man without focal neurological deficits. TSH and T3/T4 levels were all reported within normal limits. Which of the following diagnostic tests would be the most appropriate next step for this patient? A24-hour urine fractionated metanephrines and catecholamines BCT scan CGenetic testing DPlasma fractionated metanephrines

Plasma metanephrine levels dx: pheochromocytoma Which drug class is known to frequently interfere with 24-hour urine fractionated metanephrines and catecholamines? Answer: Tricyclic antidepressants. palpitations, sweating, tremor, weight loss, fever, and significant hypertension. Patients will also often complain of headaches, flushing, flank pain, and constipation. Management of hypertensive emergency in patients with pheochromocytoma is a two-step process. You must obtain alpha-blockade prior to beta-blockade. This is classically done by administering phenoxybenzamine, which is an 1- and 2-blocker, or phentolamine, a nonspecific -blocker, followed by a beta blocker, such as labetalol or propranolol. Definitive management is by surgical resection of the tumor.

A middle-aged man presents with a complaint of "my head is expanding," stating that it seems like his hat does not fit him anymore. Which of the following signs suggest a condition of excess growth hormone?

Prognathism and macroglossia dx: Acromegaly is a condition of excess growth hormone post puberty. The most common cause is a growth hormone producing pituitary gland adenoma. Symptoms include soft tissue enlargement, with macroglossia, increased ring, hat or shoe size, polyarthritis and hyperhidrosis. The pituitary mass effect can produce headaches and visual field defects. Common physical findings are frontal bossing (enlarged frontal skull bones), enlarged jaw and protruding jaw (prognathism), enlarged tongue (macroglossia), nose thickening, hirsutism, enlarged digits, oily skin and skin tags. Treatment includes somatostatin, bromocriptine and growth hormone antagonists. Acromegaly Patient presents with increased head, glove, or shoe size PE will show coarse facial features, oily skin, visual field deficits, diabetes Labs will show increased IGF-1 Most commonly caused by a pituitary adenoma Treatment is transsphenoidal resection Which conditions are commonly comorbid with acromegaly? Answer: Obstructive sleep apnea, glucose intolerance and diabetes, congestive heart failure and colon polyps or carcinoma.

A 32-year-old woman presents to the clinic for evaluation of palpitations and is diagnosed with Graves' disease. Which of the following is the most appropriate therapy for acute management of her symptoms? A 45-year-old woman with a history of Graves disease presents to the ED with a one-day history of dyspnea, fever, and agitation. Her husband believes that she stopped taking her medications one week ago. Her vital signs are BP 185/75 mm Hg, HR 141 bpm, R 24 breaths/min, and T 38.9°C. On exam, you note tremulous hands and bounding peripheral pulses. Her ECG shows a sinus tachycardia, and a chest radiograph reveals increased interstitial markings. You administer 2 L of normal saline, but her vital signs are unchanged. Which one of the following is the most appropriate therapy to administer next? A 32-year-old woman presents with tachycardia, palpitations, nausea, vomiting, and fever. She was recently diagnosed with Graves' disease. On examination, she is diaphoretic and mildly agitated with heart rate 132 beats per minute, blood pressure 189/91 mm Hg, and temperature 39.4°C. What is the correct order of treatment for this patient? What medication should be avoided in the first trimester of pregnancy when treating hyperthyroidism? Which of the following medications blocks the release of stored thyroid hormone? What medication can be given to block enterohepatic recirculation of thyroid hormone? Why are corticosteroids recommended in thyroid storm?

Propranolol Propranolol, propylthiouracil, iodide, hydrocortisone What medication should be avoided in the first trimester of pregnancy when treating hyperthyroidism? Answer: Methimazole. Which of the following medications blocks the release of stored thyroid hormone? iodine What medication can be given to block enterohepatic recirculation of thyroid hormone? Answer: Cholestyramine 4 gm oral every 6 hours. Why are corticosteroids recommended in thyroid storm? Answer: To inhibit conversion of T4 to T3, block the release of hormone from the gland, and treat relative adrenal insufficiency. an autoimmune disorder associated with thyroid-stimulating immunoglobulin antibodies directed at thyrotropin receptors (TRab) on the surface of thyroid cells Symptoms typically include tremor, tachycardia or palpitations, anxiety, weight loss, heat intolerance, pretibial myxedema, and ophthalmopathy (exophthalmos). Diagnosis is established with laboratory testing, most often revealing low thyrotropin (thyroid stimulating hormone) and elevated thyroid hormone, particularly T3, elevated thyrotropin receptor antibodies, Thyroid Storm History of thyrotoxicosis Sx: tachycardia, hyperpyrexia, agitation, anxiety PE: goiter, lid lag, hand tremor, and warm, moist skin Labs: low TSH and high free T4 or T3 Most commonly caused by an acute event (infection, trauma) Tx:Beta-blocker (propranolol) Thioamide (propylthiouracil or methimazole) Iodine solution Glucocorticoids

A 32-year-old woman with a history of Graves' disease presents to your office with a question about her medication. She just found out she is pregnant and wants to know if any changes need to be made to her treatment regimen during her pregnancy. Which of the following is the most appropriate therapy? How often should thyroid function testing occur in pregnant women with hyperthyroidism?

Propylthiouracil dx: Graves' disease How often should thyroid function testing occur in pregnant women with hyperthyroidism? Answer: Every 4 weeks. propylthiouracil (PTU) during the first trimester since it is less teratogenic than the others. Patients should be switched to methimazole during the second and third trimesters because the risk of PTU-induced liver failure outweighs any potential risks of taking methimazole.

In your primary care clinic, a 30-year-old man presents for a new patient intake history and physical examination. His BMI is 29. He has a history of tobacco use, but none currently. His paternal grandmother has type 2 diabetes. A baseline laboratory report reveals an HDL level of 45 mg/dL and a triglyceride level of 290 mg/dL. According to the American Diabetes Association, screening for type 2 diabetes mellitus in this patient is?

Recommended, based on triglyceride levels What is the DM2 screening criteria the US Preventive Services Task Force uses? Answer: Per the USPSTF, screening is recommended in adults 35-70 years of age who are overweight or obese. American Diabetes Association (ADA) recommends screening for any adult with a BMI >25 and triglycerides >250.

Which of the following is a contraindication to the use of metformin in patients with type 2 diabetes mellitus? Age greater than 70 years Insulin use Irritable bowel syndrome Renal insufficiency What agent is recommended for patients with contraindications to metformin?

Renal insufficiency contraindicated in patients with elevated serum creatinine. This means a GFR < 30 mL/min/1.73 m2. Other contraindications include congestive heart failure, radiographic contrast studies, acidosis and patients who are seriously ill. What agent is recommended for patients with contraindications to metformin? Answer: A shorter acting sulfonylurea such as glipizide.

50-year-old woman presents to her surgeon for a routine follow-up after her thyroidectomy. She reports no symptoms in the office today. Her thyroid was removed 6 months ago due to medullary thyroid cancer, and no complications were noted during the surgery. She has no other significant medical history and is on no medications. Her vitals include a BP of 122/78 mm Hg, HR of 85 bpm, RR of 12/min, T of 98.7°F, and SpO2 of 100% on room air. On physical examination, her incision is well healed, and no lymphadenopathy is present. Which of the following should be monitored to evaluate for medullary thyroid cancer recurrence?

Serum calcitonin What is the most aggressive type of thyroid cancer? Answer: Anaplastic.

A 33-year-old woman presents to her primary care office for an annual physical. The patient notes no significant medical history or history of radiation. Family history consists of diabetes and hypertension. She has no concerns or symptoms during today's visit and reports a healthy diet and exercise routine. Her vital signs are a HR of 88 bpm, RR of 17/min, BP of 118/78 mm Hg, T of 98.3°F, weight of 145 lbs, and BMI of 23.6 kg/m2. On physical exam, she is noted to have a small thyroid nodule, but otherwise, her exam is normal. What is the best next step in the management of this patient's condition?

Serum thyroid-stimulating hormone dx: incidental thyroid nodule Patients with a history of head-neck or chest-shoulder radiation or those with a known family history of thyroid cancer are at increased risk for thyroid cancer. If the TSH is low, then a radioactive iodine scan should be done to rule out a "hot nodule". All thyroid nodules need to be routinely mo

A 35-year-old woman presents to the clinic to discuss her laboratory results. The patient initially presented with amenorrhea for the past 4 months, galactorrhea, and headaches. Vital signs include a heart rate of 80 bpm, blood pressure of 120/80 mm Hg, respiratory rate of 20/minute, oxygen saturation of 98% on room air, and temperature of 98.6°F. Physical examination reveals milky nipple discharge bilaterally without palpable breast masses. Her urine pregnancy test is negative. The patient's prolactin level is 756 ng/mL. You decide to start the patient on medication to treat the suspected condition. Which of the following is the mechanism of action of the first-line medication used to treat the suspected condition?

Stimulation of pituitary dopamine receptors dx: Pituitary adenomas

A 32-year-old woman presents to the endocrinologist to discuss her most recent labs and radioactive iodine scan results. The patient was originally referred to the endocrinologist by her primary care physician after an enlarged palpable thyroid nodule was identified on her physical exam. Her vital signs are a HR of 90 bpm, RR of 18/min, T of 98.8°F, and BP of 124/82 mm Hg. On physical exam, there is a palpable thyroid nodule noted to the right superior thyroid lobe. Her most recent labs are significant for low thyroid-stimulating hormone, elevated T3, and elevated T4 levels. She recently had a radioactive iodine scan, which showed a hot nodule located to the right superior thyroid lobe. Which of the following is this patient's most likely diagnosis?

Toxic adenoma palpable thyroid nodule on physical exam and lab values consistent with hyperthyroidism: low thyroid-stimulating hormone (TSH), elevated T3, and elevated T4. The patient's symptoms would also be consistent with hyperthyroidism, including heat intolerance, moist skin, fine hair, weight loss, and if the nodule is large, they can have additional symptoms such as dysphagia, stridor, hoarseness, and dyspnea. A radioactive iodine scan would show an area of increased iodine concentration ______-----------------------____________---------------_________------- different dx: Papillary thyroid carcinoma (C) is the most common type of thyroid cancer and is most common after radiation exposure. A radioactive iodine scan would show a cold nodule.

A 22-year-old woman presents with generalized weakness, tremor, anxiety, palpitations and weight loss. Her symptoms have been constant and progressive over the last two weeks. On examination, her heart rate is 126 beats per minute, blood pressure 175/92 mm Hg and temperature 37.6C. She has lid lag, retraction, and exophthalmos noted on eye examination. What is the most likely cause of symptoms? A 52-year-old woman presents complaining of increased anxiety, palpitations, weight loss, and heat intolerance over the past 4 weeks. A physical exam is significant for a diffusely enlarged thyroid with a bruit. Radiographic and laboratory results reveal elevated radioactive iodine uptake, elevated free T4 levels, and suppressed TSH levels. Which of the following is the most likely diagnosis?

Toxic diffuse goiter The classic triad of Graves disease is a diffuse goiter, exophthalmos and pretibial myxedema. What antiarrhythmic medication can cause hyperthyroidism? Answer: Amiodarone. -------------------------------------------------------- differential dx pheochromocytoma (A) is a catecholamine-secreting tumor that arises from the adrenal medulla. Symptoms include intermittent tachycardia, hypertension, headache, and diaphoresis. pituitary adenomas (B) are a rare cause of hyperthyroidism. Patients will present not only with symptoms consistent with hyperthyroidism (e.g. tremor, tachycardia, heat intolerance) but will have symptoms related to mass effect of the adenoma including headache and visual field deficits. Toxic multinodular goiter (D) is the second leading cause of hyperthyroidism in the United States. It typically affects women older than 50 years of age and has symptoms of hyperthyroidism that are milder than those seen in Graves disease.

A 14-year old girl is accompanied by her parents for amenorrhea. She never had menses. She denies sexual activity and does not take any medications. She is a straight A student in school and plays basketball. On physical examination, her height is at the 90th percentile, weight at the 50th percentile, breasts at Tanner II, no axillary hair and no pubic hair. The rest of her examination is normal. Which of the following is the most likely diagnosis? AComplete androgen insensitivity syndrome BKlinefelter syndrome CPolycystic ovarian syndrome DTurner syndrome

complete androgen insensitivity syndrome (AIS) (formerly known as testicular femininization) genetic males appear female at birth uterus is absent. At puberty, the diagnosis is suspected by the normal development of breasts, but menstruation does not occur and sexual hair is absent. Adult heights are commensurate with those of normal males. At what penile length do you diagnose micropenis? Answer: Less than 2 cm. -------------------------------------------------------------------- Klinefelter syndrome (B) has XXY chromosomes and presents with gynecomastia, small testicles, infertility, mild delays, and a low upper-to-lower segment ratio. Polycystic ovarian syndrome (C) presents with irregular menses, acne, excess body hair, and signs of insulin resistance. Turner syndrome (D) has XO chromosomes and presents with breast buds, short stature, pedal edema, wide-spaced nipples. They may have primary amenorrhea from streak ovaries and ovarian failure.

1)Why is iodine used in the treatment of thyroid storm? Answer: In pharmacologic doses, iodine inhibits the release of T3 and T4 and inhibits the organification of iodine 2)Which medication is the preferred treatment of hyperthyroidism in the first trimester of pregnancy? Answer: Propylthiouracil (PTU). 3)What is the most common cause of endogenous Cushing syndrome? Answer: Pituitary adenoma. 4) 5) 6) 7) 8) 9)

facts- 5

1) What is the cause of ophthalmopathy in Graves' disease? Answer: Fibroblast proliferation, deposition of glycosaminoglycans, and inflammation of the extraocular muscles, orbital fat, and connective tissue. 2)What is myxedema coma? Answer: The most severe manifestation of hypothyroidism, defined by altered mental status and decompensated metabolic state. 3)What is the most common cause of Cushing disease? Answer: A benign pituitary adenoma, which secretes adrenocorticotropic hormone (ACTH). 4)What are the clinical manifestations of hyperprolactinemia? Answer: Although hyperprolactinemia in females occurs as the common triad of amenorrhea, galactorrhea and infertility, it can occur in males too, but presents with infertility, decreased libido and rarely, galactorrhea. 5)What is the treatment for primary hypoparathyroidism? Answer: While PTH analogues are used to treat osteoporosis, they are not FDA approved for hypoparathyroidism. As such, calcium and vitamin D are the mainstays of treatment. 6)At what self-monitored glucose level should patients be concerned about hypoglycemia? Answer: ≤ 70 mg/dL. 7)Is thyroid cancer more prevalent in men or women? Answer: Women are affected three times more often than men. 8) Which of the following provides the best screening for diabetes mellitus? Fasting plasma glucose 9): How often should patients be screened for diabetes mellitus? Answer: Every three years. 10)What is the name of the sign that refers to the lateral thinning of the eyebrows as seen in hypothyroidism? Answer: Queen Anne sign. 11)What is the difference between proptosis and ptosis? Answer: Ptosis is a drooping upper eyelid commonly due to the muscle weakness associated with myasthenia gravis, a different autoimmune disease (but it can occur with Graves disease). 12)Treatment of primary adrenal insufficiency generally requires daily hormone replacement with oral hydrocortisone, dexamethasone, or prednisone to prevent future adrenal crises. 13) Though primary adrenal insufficiency is usually due an autoimmune process in the United States, it is occasionally due to which infectious process world-wide? Answer: Tuberculosis may result in development of primary adrenal insufficiency. 14) 15) 16) 17) 18) 19) 20)

facts-4

searched

mechanism of action of DM drugs

A patient is diagnosed with central diabetes insipidus. Which of the following medications should you prescribe? Which of the following best characterizes diabetes insipidus?

the drug of choice is desmopressin (DDAVP), an ADH analogue. Low urine osmolality, high serum osmolality What is at risk for developing by correcting hyponatremia too rapidly? Answer: Central pontine myelinosis. Diabetes Insipidus (DI) Patient presents with polyuria and polydipsia Labs will show increase in plasma osmolality and a decrease in urine osmolality Central DI: most commonly caused by a decrease in ADH productionDiagnosis is made by vasopressin challenge test: > 50% increase in urine osmolality and decreased urine volume Treatment is intranasal DDAVP to reduce nocturia Nephrogenic DI: history of taking lithiumMost commonly caused by kidney unresponsiveness to ADHDiagnosis is made by water deprivation test: no change in urine osmolality Treatment is HCTZ, amiloride, indomethacin


Conjuntos de estudio relacionados

History Midterm 2 American Yawp Ch.10-13

View Set

Biology- Movement across the Cell Membrane

View Set

Marketing Test #1 (Chapter 1 & 2)

View Set

OSU Biology 1113 Final Exam - Mackey

View Set

Taniguchi: Slife: Chap 2-5 Vocab

View Set

Chapter 6 Section Review Questions

View Set